Vermutung im Zusammenhang mit Kolmogorov 0-1 Gesetz (für Veranstaltungen)


8

Sei (Ω,F,P) ein Wahrscheinlichkeitsraum. Vermutung:

Angenommen, wir haben Ereignisse A1,A2,...st  Anσ(An,An+1,...) , P(A)=0 oder 1 . Es gibt eine unabhängige Folge von Ereignissen B1,B2,...st

τAn:=nσ(An,An+1,...)=nσ(Bn,Bn+1,...):=τBn

Ist das wahr?


Ich denke, es gibt eine Funktion f:NN st Af(n) sind unabhängig, so dass wir wählen können Bn=Af(n). Ist das wahr? Warum Warum nicht? Wenn nicht, wie kann ich die obige Vermutung sonst beweisen oder widerlegen? Wenn es wahr ist, kann es meiner Meinung nach bewiesen werden, indem der Beweis des Kolmogorov 0-1-Gesetzes (für Ereignisse) geändert wird.


Vielleicht ist eine dieser Teilfolgen von Mengen unabhängig:

An

A2n,A2n+1

A3n,A3n+1,A3n+2

Amn,Amn+1,Amn+2,...,Amn+(m1)

Ich denke wir haben das

τAn=τAmn+i:=nσ(Amn+i,Am(n+1)+i,...)

wobei und i { 0 , 1 , 2 , . . . , m - 1 } .mNi{0,1,2,...,m1}


Es scheint, dass wir ein solches brauchen , wenn es existiert, um die folgende Bedingung zu erfüllen:f(n)

(**)σ(Af(n),Af(n+1)...)σ(An,An+1,...)

was ich denke ist wahr, wenn (und nur wenn?) f(n)n .


Andere mögliche Kandidaten für :f(n) (Angenommen, die Variablen sind st ist erfüllt. Falls erforderlich, ( ) oder f ( n ) n .)f:NN()f(n)n

  1. i=0maini

  2. 2n,3n,...

  3. i=1mbicin

  4. (ich denke t > e 1 / e )tn,tnt>e1/e

  5. i=1mbicin,i=1mbicin

  6. linear combination of trigonometric functions,linear combination of trigonometric functions

  7. Some linear combination of the above,Some linear combination of the above


Unter der Annahme, dass die Vermutung wahr ist , ist es vermutlich nicht notwendig, zu finden , das für alle möglichen Folgen von Ereignissen A 1 , A 2 , . . . weil ein solches f ( n ) möglicherweise nicht einmal existiert.f(n)A1,A2,...f(n)


Um die Vermutung zu widerlegen : Ich denke, wir müssen zeigen, dass eine solche Sequenz , die unabhängig ist, impliziert, dass B n Schwanz niemals gleich A n Schwanz ist, da B n Schwanz nach dem Kolmogorov 0-1 Gesetz (für Ereignisse) P - trivial sein wird.BnBnAnBnP

Etwas , das helfen könnte: können wir zeigen , dass oder 1 und n N , A f ( n ) , A f ( n + 1 ) , . . Anσ(Af(n),Af(n+1),...),P(A)=01ist nicht unabhängig, aber ich bin nicht ganz sicher, ob die Vermutung widerlegt ist, weil wir einige B ns konstruieren könnten, die aussehen wie:nN,Af(n),Af(n+1),...Bn

  1. Bn=An+1An
  2. Bn=AnAn1,A0=
  3. Bn=mAmn
  4. Bn=mAmn
  5. B2n=mAmn,B2n+1=mAmn
  6. Bn=lim supmAmn
  7. Bn=lim infmAmn
  8. B2n=lim supmAmn,B2n+1=lim infmAmn

Natürlich nicht zu sagen, dass eines dieser τ A n = τ B n erfüllt, aber dass B n nicht in der Form A f ( n ) vorliegen muss .BnτAn=τBnBnAf(n)


Borel-Cantelli:

  1. Wenn . Daher ist B m = lim sup A m n unabhängig.nP(An)<0=P(lim supAn)=P(lim supAmn) mNBm=lim supAmn

  2. Wenn , dann vielleicht diese Erweiterung von Borel-Cantelli ? Ich bin mir nicht ganz sicher, ob ich es verstehe oder wie es hilfreich wäre. Ich glaube nicht, dass wir etwas schließen können, wenn wir P ( lim sup A n ) haben .nP(An)=P(lim supAn)

  3. Dann gibt es den Fall von aber die früheren Bedingungen sind nicht erfüllt.nP(An)=


1
Vielleicht ein Beweis durch Konstruktion, bei dem ? B1=A1,B2=A2A1,
Jbowman

1
Für mich ist es unwahrscheinlich, dass diese Vermutung zutrifft, es sei denn, Sie fügen zusätzliche Bedingungen hinzu oder Sie meinen, dass die beiden Vervollständigungen der Algebra übereinstimmen (was fast trivial gilt). Ich kann jedoch kein Gegenbeispiel sehen. σ
P. Windridge

1
Auf jeden Fall denke ich, dass Sie mit der (einfacheren) Frage beginnen können: "Sei ein Wahrscheinlichkeitsraum. Angenommen, GF ist eine zählbar erzeugte σ- Algebra und P ( A ) = 0 oder 1 für jeden Fall A G . gibt es eine Folge unabhängiger Ereignisse B 1 , B 2 , ... in F mit Schwanz σ -Algebra G ?(Ω,F,P)GFσP(A)=01AGB1,B2,FσG
P.Windridge

2
Eine Algebra G wird zählbar erzeugt, wenn F 1 , F 2 , st G = σ ( F 1 , F 2 , ) existiert . Es ist einfach, Beispiele zu finden, bei denen die Schwanz- σ- Algebra nicht zählbar erzeugt wird. σGF1,F2,G=σ(F1,F2,)σ
P. Windridge

2
Allgemeiner kann eine Sub- Algebra einer zählbar erzeugten σ- Algebra selbst nicht zählbar erzeugt werden! Schauen Sie sich Übung 1.1.18 in math.mit.edu/~dws/175/prob01.pdfσσ
P.Windridge

Antworten:


7

Wenn Sie Ereignisse , die auf interessante Weise unabhängig sind (nicht einfach, weil P ( B n ) = 0 oder P ( B n ) = 1 ), dann ist die Vermutung falsch.BnP(Bn)=0P(Bn)=1

Hier ist ein pedantisches Beispiel. Angenommen, ist ein geeignet reicher Wahrscheinlichkeitsraum. (Ω,F,P)

Lassen sein P - Null, dh P ( A ) = 0 . Nehmen Sie A i = A , so dass die Schwanz- σ- Algebra G = { , A , A c , Ω } ist .AFPP(A)=0Ai=AσG={,A,Ac,Ω}

Beachten Sie, dass insbesondere endlich ist.G

Nehmen wir nun an, dass eine unabhängige Folge von Ereignissen ist, bei denen P ( B n ) von 0 und 1 entfernt ist . Dann wird die Schwanz- σ- Algebra H nicht zählbar erzeugt. (Siehe z. B. Übung 1.1.18 http://math.mit.edu/~dws/175/prob01.pdf , in der ein Argument wie oben beschrieben verwendet wird. Jede zählbar erzeugte P- triviale σ- Algebra hat ein Atom der Masse 1 , aber H hat kein solches Atom).B1,B2,P(Bn)01σHPσ1H

Also, ist endlich , aber H ist nicht einmal abzählbar erzeugt.GH


P(Bn)=0PσGE1,E2,GFGPEnEncBB1,1=E1B2,1=E1,B2,2=E2,,Bk,j=Ej1jk

(Bk,j)σG

GPσGσ

P(Bn)0


P(Bn)=01GτAn

H=τBn

P(Bn)=0A1,A2,σG=τ(An)Hτ(Bn)AnBnBn

Ai

AiBi
Durch die Nutzung unserer Website bestätigen Sie, dass Sie unsere Cookie-Richtlinie und Datenschutzrichtlinie gelesen und verstanden haben.
Licensed under cc by-sa 3.0 with attribution required.